🎧 New: AI-Generated Podcasts Turn your study notes into engaging audio conversations. Learn more

Loading...
Loading...
Loading...
Loading...
Loading...
Loading...
Loading...

Summary

This document covers various animal health topics, including fluid therapy in goats, respiratory issues in horses, and blood pressure monitoring. It presents case studies with associated questions, useful for veterinary science studies at secondary level.

Full Transcript

medicine 1. A 12-pound 10-day-old female goat kid is presented for watery diarrhea and weakness. Temperature is 100°F (37.5°C) [N=102-104°F, N=39.5-40.5°C] Heart rate is 180 bpm [N=90-150 bpm] Respiratory rate is 50 brpm [N=20-40 brpm] Skin tenting is 3 seconds and there is moderate enophthalmos. T...

medicine 1. A 12-pound 10-day-old female goat kid is presented for watery diarrhea and weakness. Temperature is 100°F (37.5°C) [N=102-104°F, N=39.5-40.5°C] Heart rate is 180 bpm [N=90-150 bpm] Respiratory rate is 50 brpm [N=20-40 brpm] Skin tenting is 3 seconds and there is moderate enophthalmos. The kid is approximately 8% dehydrated based on physical exam findings. You develop a fluid therapy plan but first need to estimate the fluid deficit from dehydration. What is this goat kid’s estimated fluid deficit? 0.4 liters | 1.2 liters | 120 ml | 800 ml | 960 ml 0.4 liters This goat kid needs 0.4 liters of fluid (~400 ml) to replace its dehydration deficit.The calculation is as follows: Weight in kgs x % dehydration = fluid deficit in liters; 8% dehydration=0.08 12 lbs ÷ 2.2 kg/lb = 5.45 kg 5.45 kg x 0.08 dehydration = 0.43 liters Replacement fluids should include electrolytes and bicarbonate in kids with diarrhea. Some glucose and potassium support may also be necessary. Fluids can be administered IV with an isotonic solution over 3-4 hours. Kids are notorious for chewing IV lines, so either monitor closely or use an Elizabethan collar. An alternative to isotonic IV fluids for diarrhea is IV hypertonic saline or IV hypertonic sodium bicarbonate with oral electrolytes. This only takes a few minutes and is as just as effective. Click here for a general overview of fluid replacement plans. Refs: Pugh and Baird, Sheep and Goat Medicine, 2nd ed., pp. 52-53. 2. A 10-year-old Quarterhorse gelding is presented with a 48-hour history of cough. The owner says he coughs a lot during feeding time when he gets his hay, but admits that's the only time she is in the barn to see him lately. He has a small amount of bilateral mucoid nasal discharge and is breathing heavily, especially when he exhales. Mucous membranes are pink; expiratory wheezes are audible during thoracic auscultation. Click on the labwork icon to review the hematology results. What condition is at the top of the differential diagnosis list? Bacterial pneumonia | Recurrent airway obstruction | Equine herpesvirus-1 infection | Thoracic neoplasia | Left laryngeal hemiplegia Recurrent airway obstruction The correct answer is recurrent airway obstruction (RAO). RAO is the most common disease to think of when you see older, stabled, COUGHING, WHEEZY HORSES, with increased respiratory rate at rest, without fever. Also often called "heaves," it is part of the equine asthma complex. This is a chronic inflammatory airway disease exacerbated by dust, hay, mold.Tx is primarily to DECREASE exposure to dust, hay. House OUTSIDE if possible, and bronchodilators (e.g., albuterol/ clenbuterol) plus corticosteroids. The ACVIM consensus statement on inflammatory airway disease nicely covers equine asthma. Lack of fever here makes pneumonia less likely. Lack of fever and thoracic wheezes make upper respiratory infection unlikely. Refs: Smith, Van Metre, and Pusterla's Large Animal Internal Medicine, 6th ed., pp. 578-92 and Wilson Clin Vet Advisor: The Horse, pp. 16-17. 3. Which of the following types of blood pressure monitors relies on the transfer of motion of the arterial wall to a cuff placed over a peripheral artery? Doppler | Oscillometric | Photoplethysmography | Capnograph Oscillometric An oscillometric blood pressure monitor senses the motion of the arterial wall as a blood pressure cuff deflates. The arterial wall tends to 'oscillate' up and down as pulsatile blood flow is restored, and this oscillation is transferred to the blood pressure cuff. The oscillations change as flow and pressure change. The signals are interpreted by the monitor and systolic, mean, and diastolic pressures are calculated. A Doppler blood pressure monitor detects the movement of RBCs within the vessels. Refs: Bassert, Beal and Samples, McCurnin's Clinical Textbook for Veterinary Technicians, 9th ed. pp. 856-57, 1035-37 and Thomas & Lerche Vet Anes and Analgesia for Vet Techs, 4th ed. pp 154-57. 4. Which of one of the following choices is the most important clinical issue associated with a broken blood feather in a bird? Cyst formation | Flight impairment | Infection | Hemorrhage | Permanent feather deformation Hemorrhage The blood vessel supplying a developing feather is large and damage can result in dangerous hemorrhage, especially if the feather is a large primary feather. Typically, once hemostasis is achieved, broken blood feathers are of little consequence unless damage to the follicle has occurred. Refs: The Merck Veterinary Manual online edition. 5. This dog is presented with a restrictive breathing pattern and rapid/shallow breathing. A radiograph shows severe pulmonary edema. Which conditions may cause pulmonary edema? Dilated cardiomyopathy, patent ductus arteriosus, DIC | Myasthenia gravis, idiopathic thrombocytopenia, PDA | Diaphragmatic hernia, neoplasia, blastomycosis | Vena cava syndrome, overzealous IV fluids, hyperproteinemia | Nephrotic syndrome, Spirocerca lupi, allergic bronchitis Dilated cardiomyopathy, patent ductus arteriosus, DIC Dilated cardiomyopathy, patent ductus arteriosus, and DIC can cause severe pulmonary edema like this. Note the diffuse "alveolar/interstitial" pattern--a COTTON-BALL appearance to lung parenchyma means pulmonary edema. DDX list includes NOTHING GOOD---neoplasia, congestive heart failure (CHF), overzealous IV fluids, cardiomyopathy, pneumonia (esp. inhalation of gastric contents or near-drowning), disseminated intravascular coagulation (DIC), hypoproteinemia, heatstroke and upper respiratory obstruction, to name a few. The dog in THIS radiograph has a heart-based mass (neoplasia). Follow link to see a Merck Manual image of diffuse interstitial lung pattern in a foal with acute bronchointerstitial pneumonia. Refs: Blackwell's 5-Minute Vet Consult Canine Feline, 4th ed. pp.1148-9. Image courtesy of Dr. Teri Defrancesco. 6. Which one of the following choices is a common sequela of a retained placenta in a bitch? Agalactia | Polyphagia | Uterine subinvolution | Metritis | Post-partum hemorrhage Metritis A retained placenta commonly leads to metritis in the bitch. Clinical signs associated with metritis are those of systemic illness/toxemia. Retained placenta may be identified based on client's observation of the whelping process, ultrasound, and/or persistence of a greenish-black vaginal discharge, especially if it is foul-smelling. Normal lochia is reddish brown with no odor. Refs: The Merck Veterinary Manual online edition. 7. A client presents a ewe that is heavily pregnant, due to lamb in about a week. She has not eaten well the last few days, is now recumbent and seems unaware of her surroundings. Temperature, pulse, and respiration are normal. A urine dipstick is positive for ketones. Ultrasound shows twin lambs, but heartbeats are not detectable. Which one of the following choices is the best option to save this ewe? Supportive care until the ewe goes into labor on her own | Antibiotics and anti-inflammatory therapy | Thiamine supplementation | Tube feeding | Emergency cesarean section Emergency cesarean section Emergency cesarean section is frequently necessary in ewes with severe pregnancy toxemia. If the lambs are not viable then there is no reason to delay. Removing the non-viable fetuses will help prevent further toxemia and improve the prognosis. Treatment for pregnancy toxemia depends on syndrome severity. In severe cases with a recumbent, severely depressed, and/or neurologic ewe, immediate c-section is recommended to save the dam’s life regardless of the viability of the lambs. Waiting risks all their lives because if the dam dies, so do the lambs. Aggressive fluid therapy to correct glucose, acid-base, and electrolyte abnormalities is also necessary. Severe cases often have hypocalcemia and metabolic acidosis as well as hypoglycemia. Less severe cases can be treated with fluid therapy along with induction of parturition with prostaglandins and dexamethasone. The steroid administration can help with lung maturation and increase chances of lamb survival. Unfortunately, lambs born more than seven days premature rarely survive. If caught very early, the syndrome may be resolved with supportive care, dietary changes, and sometimes oral propylene glycol. Ref: Pugh, Sheep and Goat Medicine 2nd ed. pp. 200-1. 8. An eight-month-old Quarterhorse colt has a three-week history of worsening symmetrical ataxia and knuckling in all four limbs. The colt is bright, alert and responsive, with a normal cranial nerve exam and good body condition, with no evidence of muscle atrophy in the fore or hind limbs. What disease is suspected most highly? Cervical vertebral body abscess | Botulism (shaker foal) | Congenital herpesvirus infection | Equine degenerative myelopathy (EDM) | Occipitoatlantoaxial malformation (OAAM) Equine degenerative myelopathy (EDM) Equine degenerative myeloencephalopathy (EDM) is a cause of symmetrical ataxia of all four limbs in younger horses, usually presenting in the first year of life. Thought to be associated with vitamin E deficiency in foals with a genetic susceptibility. Other causes of symmetrical tetraparesis and ataxia include cervical stenotic myelopathy (CSM, equine wobbler syndrome-young horses, 1-3 years) and occipitoatlantoaxial malformation (OAAM, esp. Arabian foals, often ataxic from birth). Equine protozoal myeloencephalitis (EPM), caused by Sarcocystis neurona (and, less commonly, Neospora hughesi ), is the principal rule out for multifocal, asymmetrical neurologic disease in horses. EPM can mimic ANY neurologic disease, but is typically characterized by asymmetric gait abnormalities, muscle atrophy, and down horses. Ref: Furr and Reed, Equine Neurology, 2nd ed. 9. Several piglets in a group weaned ten days ago in the nursery facility of a large commercial swine operation were found dead. On evaluation, some weaners have swelling around the eyes and forehead. Some are in lateral recumbency and dyspneic. Necropsy of the dead piglets reveals subcutaneous and submucosal edema. What is the most likely causative organism? Escherichia coli | Clostridium septicum | Brachyspira hyodysenteriae | Streptococcus suis | Lawsonia intracellularis Escherichia coli E. coli causes the characteristic lesions of edema disease in recently weaned piglets. Marked swelling of the periocular region, forehead and submandibular area follow infection. Piglets may die peracutely. Usually only a few piglets in a group are affected, but affected piglets perish rapidly (within 12 hours). Hemolytic E. coli that produce F18 pili and Shiga toxin 2e are implicated in edema disease. To make a definitive diagnosis, E. coli must be first isolated and then characterized as an edema disease strain (that is, producing F18 pili and Shiga toxin 2e). The course is so rapid that treatment is ineffective. Antibiotics may be administered to unaffected pigs in the group. Brachyspira hyodysenteriae causes Swine dysentery also called bloody scours. Lawsonia intracellularis causes Porcine proliferative enteritis (diarrhea, often with fibronecrotic casts). Clostridium septicum is the agent of Malignant edema in many animals. Infection occurs through contaminated wounds and turns affected muscle dark brown or black. Streptococcus suis causes septicemia and meningitis in weaners and growing pigs. Ref: Jackson and Cockcroft, Handbook of Pig Medicine, 1st ed. pp. 94-5. 10. What does a Kussmaul breathing pattern (i.e., a deep, labored respiratory pattern) most often indicate? Shock | Metabolic acidosis | Increased intracranial pressure | Hypoxemia | Pleural effusion Metabolic acidosis Kussmaul breathing (deep, labored) most often indicates respiratory compensation for metabolic acidosis. Cheyne-Stokes breathing (i.e., alternating tachypnea and bradypnea) occurs most often as a result of abnormal CO2 regulation associated with increased intracranial pressure. Bradypnea (i.e., decreased respiratory rate) can also be due to increased intracranial pressure. Restrictive breathing (e.g, fast, shallow breaths) typically indicates pleural space disease (e.g, pleural effusion) or rib fractures. Hypoxemia and shock both usually result in tachypnea in the absence of other factors that affect the breathing pattern. Refs: Bassert and Thomas, McCurnin’s Clinical Textbook for Veterinary Technicians, 9th edition, pp. 842 and Ettinger, Feldman, and Côté's Textbook of Vet Int Med, 8th ed., p. 594. 11. Which of the following presentations of a horse with strangles (e.g., Streptococcus equi subspecies equi infection) should routinely be treated with antibiotics? Intra-abdominal abscess (bastard strangles) | Depression, anorexia, and fever | Submandibular or retropharyngeal lymph node enlargement | All horses with strangles should receive antibiotics | High fever and nasal discharge Intra-abdominal abscess (bastard strangles) Antibiotics are routinely administered to those cases with internal abscesses (i.e., bastard strangles), guttural pouch empyema (infection), or those with complications. Strangles (e.g., S. equi subspecies equi infection) is a highly contagious disease in horses that causes abscesses of the submandibular and retropharyngeal lymph nodes. Horses can initially present with fever, anorexia, depression, and nasal discharge. As the disease progresses, the affected lymph nodes enlarge and eventually rupture and drain purulent exudate. If the retropharyngeal lymph nodes are affected, they can rupture and drain into the guttural pouches. Antibiotics are not typically administered to routine cases until after the abscesses have opened and drained. Refs: Bassert and Thomas, McCurnin’s Clinical Textbook for Veterinary Technicians, th edition, pp. 666. 12. What key characteristic do transmissible venereal tumors, mast cell tumors and cutaneous lymphoma have in common? Good response to cryotherapy | Intracytoplasmic granules | Typically progress to malignancy | Pleomorphism | Easy to identify cytologically Easy to identify cytologically Because they exfoliate well, round cell tumors like cutaneous lymphoma, transmissible venereal tumor and mast cell tumor are easier to identify cytologically than other skin tumors. Round cell tumors of the skin include: Mast cell tumors (MCT). Click here to see mast cell cytology. Histiocytoma. Click here to see a benign histiocytoma. Plasmacytoma. Cutaneous lymphoma. Transmissible venereal tumor. (TVT). Click here to see transmissible venereal tumor cytology. Note that malignant melanoma is often classified as a round cell tumor when its highly variable cells look round, but it is not a round cell tumor per se. Click here to see a dog with malignant melanoma on the paw, a cat with oral malignant melanoma and the same lesion, on radiograph. Ref: Cote, Clinical Veterinary Advisor-Dogs and Cats, 3rd ed. pp. 655-7, 890-1, Leiomyoma, online. images courtesy Dr. Joel Mills, Wikimedia Commons. 13. A cow showing chronic cerebral neurologic signs was euthanized and the brain was submitted for histopathology. Microscopically the brain tissue appears vacuolated. Which one of the following choices is the most likely diagnosis? Image courtesy, Dr A. Jenny Malignant Catarrhal fever | Bovine spongiform encephalopathy | Actinomycosis | Haemophilus somnus thromboembolic meninogencephalitis | Rabies Bovine spongiform encephalopathy This is the typical microscopic appearance of Bovine Spongiform Encephalopathy(BSE). The proper regulatory personnel should be contacted and definitive diagnostic tests performed. A characteristic histopathologic appearance of rabies is the presence of intracytoplasmic inclusions known as Negri bodies. Refs: Large Animal Neurology, Mayhew, 2nd Ed. p. 259. Image courtesy, Dr Al Jenny, USDA. 14. Which of the following is a correct difference between small and large bowel diarrhea in dogs and cats? Mucus in feces is common with small bowel diarrhea | Melena is seen with large bowel diarrhea | Weight loss is common with large bowel diarrhea | Hematochezia occurs with large bowel diarrhea | Tenesmus is common with small bowel diarrhea Hematochezia occurs with large bowel diarrhea Hematochezia (frank blood in the feces) is seen with large bowel diarrhea vs. melena (digested blood in the feces) is seen with small bowel diarrhea. It is important to distinguish between large and small bowel diarrhea. A thorough questioning and history from the owner can help make this distinction so the veterinarian can direct the diagnostic workup. Mucus is often seen in the feces of large bowel diarrhea, not small. The frequency of bowel movements is usually increased with large bowel diarrhea, not small. The volume of feces is frequently increased with small bowel diarrhea, not large. Tenesmus (straining during a bowel movement) is common with large bowel diarrhea, not small. Weight loss is common with small bowel diarrhea, not large. Refs: Bassert and Thomas, McCurnin’s Clinical Textbook for Veterinary Technicians, 8th edition, p. 690. 15. What is the underlying cause of nutritional secondary hyperparathyroidism ("bighead disease") in horses? Vitamin D excess | Calcium deficiency | Magnesium deficiency | Fiber excess | Phosphorus deficiency Calcium deficiency Nutritional secondary hyperparathyroidism results from an absolute or relative calcium deficiency. Typically caused by an excess of dietary phosphorus from feeds such as grass hay, grain, and bran causing hyperphosphatemia. High levels of blood phosphate lowers serum calcium levels, therefore stimulating production of parathyroid hormone and consequent bone resorption, osteoporosis, and replacement with osteoid and fibrous connective tissue (osteodystrophia fibrosa). Facial bones in particular respond with osteodystrophia fibrosa, which lends the disorder its name, "bighead disease." Enzootic calcinosis, caused by ingestion of plants, such as Cestrum diurnum, containing calcitriol-like glycosides, can result in hypervitaminosis D and excessive absorption of calcium and soft-tissue calcification. Remember that an imbalanced calcium:phosphorus ratio or inadequate dietary vitamin D can cause nutritional secondary hyperparathyroidism in iguanas. Ref: Smith, Large Animal Internal Medicine, 3rd ed. pp. 1252-23. 16. Which organism has been associated in horses with dermatitis of the face, neck, chest, and ventral abdomen; uveitis; and fibrosis and mineralization in the nuchal ligament? Habronema muscae | Culicoides spp. | Dichelobacter nodosus | Dermatophilus congolensis | Onchocerca cervicalis Onchocerca cervicalis Adult Onchocerca live in calcified nodules in the ligamentum nuchae and produce microfilaria that migrate to specific areas, including the ventral midline, face, neck, and chest. The dermatitis resulting from this is believed to be a hypersensitivity reaction to antigens from the microfilaria. Accumulation of microfilaria in the eye may also lead to conjunctivitis and uveitis. Note that Culicoides spp.(no-see-ums, biting midges) carry the onchocerca microfilariae, but do not cause the problems listed above themselves. Biting midges cause another problem sweet itch (or sweat itch, Queensland itch), a very pruritic warm weather dermatitis secondary to hypersensitivity. Rain scald, also known as streptothricosis or dermatophilosis, is caused by skin infection with the bacterium Dermatophilus congolensis. Dichelobacter nodosus, in conjunction with Fusobacterium necrophorum, is the cause of virulent footrot in sheep. Refs: Smith, Van Metre, and Pusterla's Large Animal Internal Medicine, 6th ed., pp. 1335-36. 17. Sertoli cell tumor, feminizing syndrome, and blood dyscrasias are associated with which condition? Cryptorchidism | Intersex condition | Canine transmissible venereal tumors (TVT) | Prostatic squamous metaplasia | Balanoposthitis Cryptorchidism Cryptorchidism increases the risk for Sertoli cell tumor growth, which can lead to hyperestrogenism, causing feminizing syndrome and blood dyscrasias (myelosuppression). You can also see increased risk of seminomas, testicular torsion, and benign prostatic hyperplasia (BPH). If both testicles are not in the scrotum by six months, the animal is cryptorchid. Normally testes descend by six to eight weeks in dogs, and by four months in cats. Tx is CASTRATION, discourage breeding. Balanoposthitis is inflammation of the penile or preputial mucosa. Ref: Cote, Clinical Veterinary Advisor: Dogs and Cats, 4th ed., pp. 225 and 495. 18. A commercial mink farm has a disease problem characterized by poor reproduction, gradual weight loss, oral and GI bleeding, renal failure and uremia, and high mortality. Which one of the following choices is the most likely diagnosis? Canine distemper virus | Epizootic Catarrhal Gastroenteritis | Mink viral enteritis | Aujeszky's Disease (Pseudorabies) | Aleutian disease Aleutian disease Aleutian disease in mink is chronic immunosuppressive wasting disease caused by a parvovirus. The name "Aleutian" comes from the homozygous Aleutian (blue) coat color gene of mink that are most severely affected. Note that mink get all four of the other diseases listed. Mink have a prion disease too; Transmissible mink encephalopathy (Mink Scrapie). Ref: Quesenberry and Carpenter, Clin Med and Surg of Ferrets, Rabbits and Rodents, 3rd ed. p. 135-6. 19. A 6-year-old Arabian endurance stallion is presented for muscle cramping after training. The horse is sweaty and has not cooled down properly. There is a slight audible thumping sound associated with each rapid breath. While waiting for the initial lab work results, what treatment should be given IV for the respiratory pattern described in this horse? Sodium bicarbonate | Glucose | Calcium | Insulin | Need more information Calcium Calcium. This respiratory pattern is called synchronous diaphragmatic flutter (a.k.a. “thumps") and is due to hypocalcemia; calcium should be given IV immediately. Hypocalcemia causes the phrenic nerve to be hyperexcitable, so it fires with each atrial depolarization due to its position as it courses across the pericardium. Thumps is seen not uncommonly in endurance horses, and they often have a concurrent hypochloremic, hyponatremic, hypokalemic, hypomagnesemic metabolic alkalosis due to losses in sweat. Refs: Robinson’s Current Therapy in Eq Med, 7th ed. and the Merck Veterinay Manual online edition. 20. A 2-year-old mixed breed dog is presented with an acutely enlarged eye OS. Intraocular pressure (IOP) is > 30 mm Hg You suspect glaucoma. What are some underlying pathologic causes of primary glaucoma? Ectropion, Interstitial keratitis | Dacryocystitis, Chorioretinitis | Cataracts, Persistent pupillary membranes | Primary lens luxation, hyphema | Closed filtration angle, Goniodysgenesis Closed filtration angle, Goniodysgenesis Glaucoma occurs when the normal outflow of aqueous humor is impaired. Primary glaucoma occurs from primary eye disease, like closed filtration angle or goniodysgenesis. Can also see secondary glaucoma from other ocular diseases like lens luxation, hyphema, anterior uveitis or intraocular tumors. Primary glaucoma is generally a BILATERAL disease, but often is clinically apparent only in one eye at first. Initial signs are so subtle that it is difficult to identify without measurements of intraocular pressure (IOP). Most dogs will develop glaucoma in the other eye without prophylactic treatment. Prophylactic Tx with a beta-adrenergic blocker or prostaglandin can delay the onset of glaucoma for up to 2.5 years. "Acute" glaucoma often results from a chronic problem that has finally become so severe that IOP is very high and is an emergency. If the IOP remains elevated for 24-48 hours, irreversible blindness may ensue. Dogs will present with obvious ocular pain, and the eye looks abnormal. Even if the dog is already blind in the affected eye, they should be treated to address the pain. Refs: Cote's Clinical Vet Advisor Dog and Cat 4thed. p. 389, Maggs, Miller, and Ofri's Slatter's Fundamentals of Vet Ophthalmology, 4thed., pp. 241-8, Kirk's Current Veterinary Therapy XV, pp. 1170-6 21. Many cows in a dairy herd have lost weight and show poor milk production. Examination of the withers, shoulders, and dewlap of affected cattle reveal the following findings (see below). What parasite has been found? Lice | Chorioptic mange mites | Culicoides spp. (biting midges) | Trombiculid mites (chiggers) | Argasid ticks Lice Lice. This clinical presentation is consistent with a blood-sucking louse infestation (Linognathus vituli, the long-nosed cattle louse). There are chewing lice (e.g., Damalinia bovis) and blood-sucking lice. Different lice have different site predilections on the host and tend to affect different age groups of cattle. For all types, stressors (e.g., health problems, high stocking density, poor feed quality, gestational status) contribute to susceptibility and degree of infestation. In addition to the clinical signs discussed in this case, pediculosis can cause damage to the hide and hair. L. vituli can act as a mechanical vector for Anaplasma marginale, the cause of bovine anaplasmosis. Rx: Pour-on (for biting and blood-sucking lice) or injectable (for blood-sucking lice) formulations of pyrethroids, macrocyclic lactones, or organophosphates. It is also important to effectively disinfect the premises. Refs: Haskell, Blackwell’s 5-minute Vet Consult: Ruminant,. Image courtesy of Alan R. Walker. 22. Which one of the following choices is the best way to prevent grain overload? Give penicillin when concentrates are increased | Introduce dietary changes slowly | Restrict movement and access to legume hay | Transfaunate the rumen | Ensure adequate water supply Introduce dietary changes slowly Introduce dietary changes slowly. Grain overload results when ruminants ingest too much feed with readily digestible carbohydrates at one time. Excessive carbohydrate causes a shift to gram-positive organisms in the rumen, especially Streptococcus bovis, which proliferate excessively. These organisms produce large amounts of acid which damage the rumen and are absorbed, causing acidosis, dehydration, ileus, and eventually cardiovascular collapse and death. Transfaunation of the rumen fluid is helpful after grain overload is resolved to help restore the normal flora of the rumen. Refs: Smith, Large Animal Internal Medicine 6th ed. pp. 869-871. 23. Which of the following most correctly describes the focus of the initial evaluation of a critically ill or injured patient upon arrival at the emergency room? Heart rate, temperature, respiratory rate, evaluation for pain | Airway, breathing, circulation, disability | External assessment, auscultation, pulse strength | Consciousness, mucous membrane color, perfusion time, ambulation | Attitude, appetite, urination, defecation Airway, breathing, circulation, disability The primary survey (initial assessment) of a critical patient involves evaluation of the Airway, Breathing, Circulation, and Disability (ABCD). This is similar to the protocol for cardiopulmonary cerebral resuscitation (CPR). Disability includes assessment of the level of consciousness, ability to move and use each limb, and an evaluation for pain. A fifth step is an exam for external evidence of disease/injury, such as hemorrhage, bruising, lacerations, swelling, painful areas, hernias, obvious anatomical deformity, etc. ABCDE is the complete acronym for the primary survey. Necessary emergency therapy is instituted prior to the secondary survey, which includes a complete physical exam. Refs: Silverstein & Hopper, SA Critical Care Med, pp. 5-9, McCurnin's Clin Textbk for Vet Techs, 8th ed. pp. 908-13. 24. In addition to daily insulin injections, which of the following interventions form the foundation of treatment for non-ketotic diabetes mellitus in dogs (and cats)? Glucotrol®; high calorie/low fiber diet; promote water consumption | Antidiuretic hormone; Florinef®; water restriction | Omeprazole; low sodium/highly digestible diet; free choice water | Glipizide; restrict activity; urinary acidifiers | Balanced diet; exercise; treat concurrent diseases Balanced diet; exercise; treat concurrent diseases The foundation of uncomplicated diabetes mellitus (DM) management is insulin, a balanced diet (BID), exercise (decreases insulin requirement) and treatment of concurrent diseases. Diet formulations will depend on the animal's body condition, species and concurrent diseases. In dogs, diets higher in fiber are generally recommended. In cats, high-protein, low- carbohydrate diets are preferred. Note - glipizide and Glucotrol® are the same oral hypoglycemic drug sometimes used to Tx type 2, NON-insulin dependent DM in cats. Not a substitute for insulin therapy - recommended only when owners will not give insulin injections and/or may be considering euthanasia. Hoofbeats hint: -Middle-aged -PU/PD, and -Eating a lot (but losing weight), is highly suggestive of "endocrine" disease, especially "diabetes mellitus". Refs: Cohn and Cote, Clinical Veterinary Advisor-Dogs and Cats, 4th ed. pp. 252-3, AAHA Diabetes Guidelines 2018 for dogs and cats (PDF) and AHAA Guidelines webpage. 25. A local stable has an outbreak of respiratory disease among the younger foals (conjunctivitis, nasal discharge, anorexia, submandibular edema). Four months later, three mares from the same farm are presented for late-term abortions of fresh fetuses. The mares appear clinically normal otherwise. What disease is the most concerning? Equine rhinopneumonitis | Equine coital exanthema | Streptococcus equi spp. zooepidemicus | Equine viral arteritis | Contagious equine metritis Equine rhinopneumonitis If you hear "nasal discharge" in younger horses and then abortion storms four to five months later in late pregnant mares, think equine herpesvirus-1 (EHV-1, equine rhinopneumonitis). Vaccination of pregnant mares at five, seven, and nine months gestation helps prevent abortion. Refs: Smith, Van Metre, and Pusterla's Large Animal Internal Medicine, 6th ed. pp. 569-71, 573-75. 26. Which one of the following agents should NOT be used when electrical defibrillation is performed on a patient with cardiac arrest? KY Jelly | Chlorhexidine solution | Alcohol | Contact gel Alcohol Alcohol is flammable and if present on the skin during the electrical shock used for defibrillation, explosion or fire may result. No one should contact the dog or the table during defibrillation as they may be shocked as well. Cardiac arrhythmia can result in shocked humans. The person in charge of the defibrillator must announce their intention by calling out 'clear' or another warning, prior to applying the electrical shock. Ref: McCurnin’s Clin Textbk for Vet Techs, 9th ed., p. 854. 27. The following object is seen while doing a routine fecal exam on a dog. What treatment might be indicated? Diethylcarbamazine citrate monthly | Sulfadimethoxine for all pets, skin scrape owner | Amitraz dip weekly | Flea control, pet and environment | Elizabethan collar, prevent coprophagia, Clindamycin Flea control, pet and environment Start flea control on both the pet and environment. These are tapeworm eggs (Dipylidium caninum), carried in the intestines of fleas. Although drugs like fenbendazole or praziquantal can treat the tapeworms, typically need to address the underlying flea infestation to get rid of the problem. The owner may report, or you may see segments of the mature tapeworm called proglottids stuck to the hair around the rear end of a cat or dog like flat pieces of rice. Can also see flea larvae or the black spirals of flea larvae feces in the coat. Refs: Blackwell's 5-Min. Vet Consult Canine-Feline, 4th ed. p. 1322 and The Centers for Disease Control DPDx Parasite Image Library-Dipylidium Dr. Joel Mills, Wikimedia commons. 28. A client calls to discuss her mature, male neutered, indoor cat’s diet. She saw something online about benefits of feeding him raw trout instead of commercial cat food. What is a key point to communicate? Fish is a good protein source for cats | It is high in vitamin A, which can be dangerous | It is considered a hypoallergenic diet | Would likely need to supplement phosphorous and magnesium | Contains thiamine antagonists Contains thiamine antagonists Raw freshwater fish may contain high levels of thiamine antagonists, which can cause deficiency by quickly decreasing thiamine in the diet. Fish is not actually deficient in thiamine. Cervical ventroflexion can occur in cats with thiamine deficiency, as well as other neurologic signs like blindness, ataxia, weakness, tremors, seizures and coma. GI signs typically occur first. If left untreated, this can be fatal. Fish can be allergenic, leading to increased risk of atopic dermatitis. They can also contain high levels of mercury, which can lead to neurological disturbances. Fish can contain high concentrations of calcium and phosphorous. Cats require more vitamin A in their diets than do other mammals because they cannot convert beta-carotene into vitamin A. Refs: Cote, Clinical Veterinary Advisor - Dogs and Cats, 4th ed., pp. 968-9. 29. A seven-month old male German shepherd is presented who recently stumbled and fell down some stairs. In the last three to four weeks the owner has noted some difficulty rising, reluctance to climb stairs, bunny hopping gait and occasional hind limb lameness. The dog is weight-bearing today. On physical examination there is positive Ortolani sign on the right. What is the diagnosis? Panosteitis | Hip dysplasia | Hypertrophic osteopathy | Hypertrophic osteodystrophy | Multiple cartilaginous exostoses Hip dysplasia This is hip dysplasia. Think LARGE, LAME dogs, especially German shepherds. Follow these links to see radiographs of: early hip dysplasia,(no DJD yet), severe hip dysplasia (Note flattened, angular femoral head, poor congruence (parallel line) with acetabulum) and, in contrast, normal canine hips with deeply seated femoral heads and good congruence with acetabulae. Coxofemoral luxation (dislocated hip) commonly occurs after more severe trauma (ie: hit by car), but presents NON-weight- bearing. Panosteitis is definitely a R/O in a young, lame german shepherd, but is characterized more by a shifting leg lameness with long bone pain, not hip laxity. Refs: Blackwell's 5-Min. Vet Consult Canine-Feline, 4th ed. pp. 624-25 and Wikipedia. 30. What kind of heart murmur is most often associated with the condition seen in this necropsy specimen from a cat? Systolic murmur, right side, tricuspid valve | Systolic murmur, left side, mitral valve regurgitation | Diastolic murmur, right side, pulmonic valve | Machinery murmur | Diastolic murmur, left side, aortic valve regurgitation Systolic murmur, left side, mitral valve regurgitation This is feline hypertrophic cardiomyopathy (HCM) the most common heart disease of cats. Often, cats with HCM are asymptomatic and the problem is discovered on physical exam, with auscultation of a systolic murmur (left AV, mitral valve), evident in 80% of cats with HCM. Physical signs in cats with clinical HCM may include tachypnea, dyspnea, anorexia, vomiting and lethargy. Ref: Cote, Clinical Veterinary Advisor-Dogs and Cats, 4th ed. pp. 82-83, 90. Image courtesy of Dr. Terri DeFrancesco. 31. What drugs are indicated to treat a cat that is sick with Toxoplasma gondii? Clindamycin or Sulfadiazine | Amprolium and Tobramycin | Pradofloxacin or Doxycycline | Griseofulvin and Metronidazole | Metapyrone and Ketoconazole Clindamycin or Sulfadiazine Treatment for toxoplasmosis is typically clindamycin or a sulfadiazine, pyrimethamine combination. Think of Toxoplasma gondii (with neurologic and ocular manifestations) in association with feline immunodeficiency virus (FIV). AVOID griseofulvin in FIV- infected cats (or use with extreme caution). May induce severe neutropenia. AVOID use of MLV vaccines also- they may induce disease in immunosuppressed cats. Refs: Blackwell's 5-Min. Vet Consult Canine-Feline, 4th ed. pp. 490-1, 1350-1 and Cote, Clinical Veterinary Advisor-Dogs and Cats, 4th ed. pp. 325-7. 32. Which animals are predisposed to occipitoatlantoaxial malformation (OAAM)? Male cats, Dorset sheep | Dolichocephalic dogs, Saanen goats | Toy breed dogs, Arabian foals | Siamese and Maine coon cats, Guernsey cattle | Giant breed dogs, Standardbred foals Toy breed dogs, Arabian foals Equine occipitoatlantoaxial malformation (OAAM) is due to a congenital C1-occipital malformation, particularly in Arabian foals. Look for foals that are ataxic from birth or shortly thereafter. In dogs, atlantoaxial subluxation is most commonly a congenital problem of young toy or miniature breeds. Occasionally occurs large breeds, like Rottweilers and Doberman Pinschers. Click here to see a radiograph of OAAM in a dog. The other big congenital cervical malformation in horses is cervical vertebral stenotic myelopathy (CVSM, equine wobbler syndrome, also called cervical vertebral compressive myelopathy or CVCM), caused by stenosis of cervical vertebral canal. Look for younger horses (1-3 years) with normal mentation, but pronounced hindlimb ataxia, a clumsy "tin soldier" gait, knuckling and stumbling. You can make the hind end sway when pulling on the tail (positive tail test, suggesting paresis). Exacerbation of symmetric signs with elevation of the head, lack of cranial nerve signs, atrophy or systemic illness in a young horse suggests CVSM. Ref: Smith, Van Metre, and Pusterla's Large Animal Internal Medicine, 6th ed. p. 1094. 33. What is the presumptive diagnosis in a chicken with the following necropsy finding? Click here to see image Click here to see image Marek's disease | Infectious bursal disease | Histomoniasis | Renal dystrophy | Avian spirochetosis Infectious bursal disease This is infectious bursal disease. Note the enlarged, balloon-like hemorrhagic bursa of Fabricius. Caused by a birnavirus, shed in feces and transferred barn to barn via fomites. Very stable. Difficult to eradicate from premises. Signs include clinical picture given above in older birds, or MORE IMPORTANT subclinical form in young birds, which causes immunosuppression via destruction of immature lymphocytes in bursa, thymus, spleen. Immunosuppressed birds do not respond well to vaccination and are predisposed to infections with normally nonpathogenic viruses and bacteria. Common disease usually exacerbated by IBDV infections. See large economic losses. Ref: The Merck Veterinary Manual online edition. 34. What is being demonstrated in the snakes in these images? Click here to see image Click here to see image Evaluation of cloacal development | Pregnancy test | Relieving constipation | sex determination | Measurement of approximate age sex determination This image demonstrates sex identification in snakes using a probe; the middle snake is a male. Click here to see the labeled image. The sex of snakes is determined by probing for the hemipenes with a blunt, lubricated probe. In males, the probe will enter the hemipenes (depth of 6–14 subcaudal scales). In females, it enters the cloacal gland (depth of 3–6 scales). Refs: Meyer and Donnelly, Clinical Veterinary Advisor: Birds and Exotic Pets. 35. Outbreaks of equine respiratory disease, abortion, and neurologic disease can all be caused by infection with which one of the following choices? Equine influenza virus | Equine herpesvirus-1 | Equine viral arteritis | Equine coital exanthema | Potomac horse fever Equine herpesvirus-1 Infection with equine herpesvirus-1 can cause respiratory disease, abortion, and neurologic disease. The respiratory disease caused by equine herpesvirus-1 and herpesvirus-4 is clinically indistinguishable. Equine herpesvirus-2 is typically associated with respiratory disease/fever in young animals but not with abortion or neurologic disease. Equine influenza virus causes outbreaks of high fever, hacking cough, and occasionally myopathy in susceptible horses. Equine arteritis virus causes outbreaks of respiratory disease/fever and abortion but not neurologic disease. Refs: Smith, Large Animal Internal Medicine, 3rd ed. pp. 511-2. 36. Which parasite is associated with tumor-like stomach nodules and "summer sores" (skin lesions) in horses? Gastrophilus spp., Ollulanus tricuspis | Parascaris equorum, Anoplocephala magna | Physaloptera spp., Spirocerca lupi | Draschia megastoma, Habronema spp. | Strongylus vulgaris, Cyathostominae Draschia megastoma, Habronema spp. Habronema spp. (Habronema muscae, H. microstoma, and Draschia megastoma) in horses can cause tumor-like stomach nodules and sometimes cutaneous lesions ("summer sores"). Strongylus vulgaris (large strongyles), Cyathostominae (small strongyles), Parascaris equorum, and Anoplocephala magna (tapeworm) are all intestinal parasites. Spirocerca lupi (esophageal worm) is a disease of dogs in the southern U.S. and tropical climates. Ollulanus tricuspis is an uncommon gastric parasite of cats. Refs: Smith, Van Metre, and Pusterla's Large Animal Internal Medicine, 6th ed., p. 1337 and Cote, Clinical Veterinary Advisor - Dogs and Cats, 4th ed. pp. 933-5. 37. Which one of the following choices is the correct definition of lymphangiectasia? Dilated lacteals | Lymphoid hyperplasia | Lymphatic vessel obstruction | Lymphadenopathy | Increased production of lymphocytes due to inflammation Dilated lacteals The definition of lymphangiectasia is the pathologic dilation of lacteals that absorb fats into the lymphatic vessels. The small white spots on the image are dilated lacteals in the duodenum. Lymphangitis is inflammation of lymphatic vessels. Refs: Blackwell's 5-Min. Vet Consult Canine-Feline, 4th ed. pp. 250-1 and Côté, Clinical Veterinary Advisor-Dogs and Cats, 3rd ed. pp. 606-9. 38. What feline condition may cause the lesion evident in this image? Subaortic stenosis | Ventricular septal defect | Restrictive pericarditis | Endocarditis | Hypertrophic cardiomyopathy Hypertrophic cardiomyopathy This is feline hypertrophic cardiomyopathy (HCM) the most common heart disease of cats. Often, cats with HCM are asymptomatic and the problem is discovered on physical exam, with auscultation of a systolic murmur (left AV, mitral valve), evident in 80% of cats with HCM. Physical signs in cats with clinical HCM may include tachypnea, dyspnea, anorexia, vomiting and lethargy. Some cats may present with hindlimb paresis or paralysis, cyanotic nail beds, toe pads and cold extremities, decreased or absent pulses, contracted and painful hindlimb muscles and relentless crying due to aortic thromboembolism. Ref: Cote, Clinical Veterinary Advisor-Dogs and Cats, 4th ed. pp. 82-83, 902. Image courtesy of Dr. Terri DeFrancesco. 39. A two-year-old ferret is presented that looks like the image below. What is the clinical diagnosis? Click here to see image Click here to see image Hyperestrogenism | Insulinoma | Aleutian disease | Hypoadrenocorticism | Autoimmune hemolytic anemia Hyperestrogenism Extremely pale skin and a swollen vulva in an unspayed female ferret is estrus-induced aplastic anemia (also known as hyperestrogenism) until proven otherwise. Majority of ferrets are spayed or neutered before six weeks because females are induced ovulators and can develop severe hyperestrogenism if not bred. Insulinomas are common in ferrets two to three years of age. Look for weakness, lethargy, posterior paresis, drooling, teeth-grinding, and seizures (due to severe hypoglycemia). The most common clinical sign of hyperadrenocorticism (HAC) is HAIR LOSS beginning on tail/rump, progressing flank and head. In females, may see a swollen vulva and enlarged mammillae. Males become aggressive and develop stranguria secondary to prostatic enlargement. With HAC, enlarged adrenal glands are often palpable cranial to kidneys. HAC can also cause hyperestrogenism and bone marrow suppression. May be more appropriate to refer to this disease in ferrets as "adrenocortical disease" rather than "hyperadrenocorticism" since pathogenesis and clinical signs are different than hyperadrenocorticism (Cushing's disease) in dogs. Aleutian disease causes an immune-complex glomerulonephritis and arteritis (seen more in MINK). Ref: Quesenberry and Carpenter eds, Ferrets, Rabbits, and Rodents: Clinical Medicine and Surgery, 2012, pp. 86-102. 40. Which of the following diseases can cause abortions, neurologic disease, and upper respiratory tract disease in horses? Equine herpesvirus | Streptococcus equi subspecies equi | Equine infectious anemia | Equine viral arteritis | Equine influenza virus Equine herpesvirus Equine herpesvirus is highly contagious via aerosolization, respiratory secretions, fomites, and aborted fetuses. It can cause abortions in the seventh to eleventh month of gestation. Equine herpesvirus can also cause upper respiratory tract disease that looks very similar to other upper respiratory tract infections with fever, anorexia, lethargy, and serous nasal discharge. Lastly, it can less commonly cause neurologic disease, with several large outbreaks reported in the US in the last few years. Refs: Bassert and Thomas, McCurnin’s Clinical Textbook for Veterinary Technicians, 9th edition, pp. 665-666. 41. A four-year-old male beagle is presented with straining and crying when attempting to urinate. Only drops of bloody urine are produced. A lateral abdominal radiograph reveals the presence of stones in the bladder and the urethra. The os penis is pictured below. Which one of the following choices is the next best step in this case? Abdominal ultrasonography | Schedule cystotomy later in the week | Flush stones back into bladder | Try to express the urinary bladder | Perform a pyelogram Flush stones back into bladder Try to flush the stones back into bladder. This is an emergency situation, as urinary obstruction caused by urolithiasis is potentially life-threatening. Uroliths found at the level of the os penis can be extremely difficult to dislodge; a urethrotomy is necessary in some cases. Dissolution of some types of stones is possible with medical therapy and a special diet, after the obstruction is relieved. Cystotomy is sometimes necessary when obstruction cannot be relieved via catheterization and flushing. It is also required to remove bladder stones that cannot be dissolved medically. Ref: Cote, Clinical Veterinary Advisor-Dogs and Cats, 3rd ed. pp. 1032-4. Image courtesy, Dr Kalumet. 42. This 5-day-old calf was born normally to a primiparous dam, but was found down today. The animal is stiff, has opisthotonos, and is hyperreactive to tactile, visual, and auditory stimuli. There is no visual deficit, but the calf has inducible nystagmus which varies with changes in head position. Movement of the neck triggers tonic clonic convulsions. Rectal temperature is 105.5°F (40.8°C).. [N=101.5-103.5°F, N=37.8-39.7°C]. Which one of the following choices is the most likely diagnosis? Neosporosis | Nervous coccidiosis | Bacterial meningitis | Polioencephalomalacia | Bovine spongiform encephalopathy Bacterial meningitis Stiffness, inducible nystagmus, and intermittent tonic-clonic convulsions in a 5-day-old calf are indicative of bacterial meningitis. These cases are hyperreflexive and extremely hyperesthetic. Although this patient is still visual, blindness is often seen. The fever is also typical of bacterial meningitis, and may be exacerbated by the seizures. The causative agent is usually the bacterial organism E. coli which accesses the CNS hematogenously after entrance through the tonsils, especially in colostrum-deprived calves. Failure of passive transfer is more common in calves born to primiparous dams. A lumbosacral CSF tap can be performed to confirm the diagnosis. Nervous coccidiosis is a disease of weaned calves approximately 3 to 5 months of age. The affected animals are usually comatose and non-visual. This calf is young for polioencephalomalacia (PEM), and is not blind, a common clinical sign in PEM. If the calf had neosporosis it might resemble this patient, but wouldn't have a fever or hyperesthesia, and probably would have been born with the condition. Refs: Divers and Peek, Rebhun's Diseases of Dairy Cattle, 2nd ed. pp. 508-10 and Smith, Large Animal Internal Medicine, 3rd ed. pp. 896-900. Image courtesy Dr. Lisle George copyright 2012. 43. Which of these organism choices has a reproductive cycle in cats ? Toxoplasma gondii | Brachyspira hyodysenteriae | Mycobacterium avium subspecies paratuberculosis | Neorickettsia helminthoeca | Haemophilus influenzae Toxoplasma gondii Toxoplasma gondii has a reproductive cycle in cats. Toxoplasmosis is a multisystemic protozoal disease that affects primarily young animals and the immunosuppressed (Think cats with FIV, human AIDS patients). T. gondii is an important cause of abortion in sheep, goats and sometimes pigs. Refs: Bassert, Beal and Samples, McCurnin's Clinical Textbook for Veterinary Technicians, 9thed. pp. 116. 44. When the "quick" is accidentally cut during a nail trim, which one of the following choices can be used to stop the bleeding? Acetylsalicylic acid | Silver nitrate | Pyrethrin | Acid citrate dextrose Silver nitrate Silver nitrate is used for chemical cauterization to stop bleeding. Refs: Bassert and Thomas, McCurnin's Clinical Textbook for Veterinary Technicians, 8th ed. p. 819. 45. A litter of five-week-old shelter kittens is presented with red swollen eyes for the past week. On exam, kittens have purulent ocular discharge, marked chemosis and blepharospasm. What is the most likely diagnosis? Chlamydiosis | Feline calicivirus | Panleukopenia | Bordetellosis | Rhinotracheitis Chlamydiosis Chlamydia felis infections are characterized by serous to purulent ocular and/or nasal exudate, and marked chemosis. Rhinotracheitis, caused by feline herpesvirus, typically causes serous ocular and/or nasal discharge with possible ulcerative keratitis. Later secondary bacterial infections may lead to purulent exudate. Bordetella bronchiseptica in cats may cause coughing or even pneumonia. Panleukopenia virus, a parvovirus of young unvaccinated kittens, causing bone marrow suppression and gastrointestinal disease. Ref: Cote, Clinical Veterinary Advisor-Dogs and Cats, 4th ed. pp. 157-59. 46. A small kitten is presented with an open wound in the right ventral neck containing a 1.2 inch (3 cm) long parasite covered in tiny black spines. What is the organism? Horse bot (Gasterophilus) | Horse stomach worm larva (cutaneous habronemiasis) | Sheep nose bot (Oestrus ovis) | Rabbit bot (Cuterebra) | Cattle grub (Hypoderma) Rabbit bot (Cuterebra) The rabbit botfly larva Cuterebra is occasionally found in outdoor-roaming cats and dogs. In North America, cases are usually seen during late summer and fall. The fistula serves as a breathing hole for the larva which eventually exits, pupates and becomes a fly. The normal hosts are rabbits, squirrels, chipmunks and mice. Cuterebra flies lay eggs near the nests, burrows, and traffic pathways where they can attach to the body of hosts. The eggs hatch in response to body heat and larvae enter the body via the mouth or nostrils during grooming, but may also enter through a wound. They penetrate the tissues and travel to specific subcutaneous areas where they mature. Extract the larva carefully in one piece to avoid a severe local inflammatory reaction that can result from a ruptured grub. The area should NOT be squeezed to avoid damage to the larvae. Anaphylaxis has also been reported anecdotally. If there is only a swelling and no obvious breathing hole yet, an abscess or foreign body are ruleouts for this lesion. Cattle grubs (Hypoderma spp.) are generally lighter colored with less prominent spines and rarely found in pets. The larvae that cause cutaneous habronemiasis are tiny - they emerge from infected flies feeding on wounds and invade the damaged tissue. Horse bots (Gasterophilus spp.) live in the stomach and are not known to infect dogs and cats. Sheep nose bots (Oestrus ovis) are smaller than cuterebrae larva. They live in the nares and only affect sheep and goats. Refs: Medleau & Hnilica's Small Animal Dermatology: A Color Atlas and Therapeutic Guide, 2nd pp. 128-9, Cote's Clinical Vet Advisor, Canine and Feline, 4th ed. p. 694. Images courtesy of Dr. Shirley Scott. 47. Which organism causes "flabby bag" mastitis in cows, also called "Milk Drop Syndrome", because of a sudden drop in milk production? Escherichia coli | Klebsiella spp | Staphylococcus aureus | Mycobacterium bovis | Leptospira sv. Hardjo Leptospira sv. Hardjo Leptospira causes "flabby bag mastitis" in cattle, so called because the milk production can drop so fast the udder becomes soft and flabby. Severe clinical mastitis in cattle can be caused by coliforms like E. coli and Klebsiella. According to the Merck Vet Manual, 11th ed., prognosis for Klebsiella infection should be particularly guarded, as cows are twice as likely to be culled or die than those infected by other coliforms. Refs: Smith, Van Metre, and Pusterla’s Large Animal Internal Medicine, 6th ed., pp. 913-14. 48. A silicone gastrostomy tube can typically be used for what period of time? 2 to 4 weeks | 3 to 7 months | 5 to 8 weeks | 8 to 12 months 8 to 12 months Gastrostomy tubes made of silicone can last 8-12 months. Latex tubes do not last more than about 12 weeks as the latex eventually breaks down from exposure to stomach acids. A gastrostomy tube is a feeding tube that passes through the abdominal wall into the stomach. These allow placement of food directly into the stomach and are used when the patient is unable or unwilling to eat normally because of damage to the esophagus, head, or neck. Often called "PEG" (percutaneous endoscopic gastrostomy) tubes because they are placed using an endoscope. Ref: McCurnin and Bassert's Clin Textbook for Vet Technicians, 7th ed pp. 618-21. 49. An obese six-year-old neutered male Pomeranian presents with a two-year history of intermittent dry, honking cough. This morning he started retching. The dog never boards in a kennel and has not traveled recently. On physical exam, a cough can be elicited by pressing on the trachea. A heart murmur is noted, loudest on the left at the 5th-6th intercostal space. The rest of the exam was unremarkable. What diagnosis is at the top of the differential list? Infectious tracheobronchitis | Chronic bronchitis | Laryngeal paralysis | Congestive heart failure | Tracheal collapse Tracheal collapse When you hear a "HONKING COUGH" and heart murmur in a fat little dog that coughs after touching his tracheal area, think tracheal collapse. Toy/Min. breeds (Pomeranians, Toy poodle, Min. poodle, Yorkies) are predisposed. Mitral valve insufficiency / dysplasia murmurs are often detected concurrently. Intermittent hx and lack of fever suggest a chronic, non-infectious cause. For definitive diagnosis do thoracic radiographs, bronchoscopy. Refs: Blackwell's 5-Minute Vet Consult Canine Feline, 4th ed. pp. 904-6, 1352-3. 50. 4-Ipomeanol toxicity (moldy sweet potato) is indistinguishable from what other bovine respiratory disease? Enzootic nasal granuloma | Contagious bovine pleuropneumonia | Tracheal edema syndrome | Chronic obstructive hypertensive cor pulmonale | Acute bovine pulmonary emphysema and edema Acute bovine pulmonary emphysema and edema 4-Ipomeanol toxicity (moldy sweet potato) is indistinguishable from acute bovine pulmonary emphysema and edema (ABPEE, also known as fog fever). Associated with L-tryptophan in lush pastures, which is converted into pneumotoxic compounds that damage respiratory epithelial cells. Other diseases that present similarly are moldy sweet potato toxicity and feedlot acute interstitial pneumonia. Refs: Smith, Large Animal Internal Medicine 6 th ed. p. 680-682. 51. It is July in Louisiana. An adult male neutered barn cat with exposure to horses is presented for lameness of at least a week’s duration. After sedation, a wound is visible under the clipped hair of the affected limb. This is most likely an example of what problem? Aberrant Culicoides spp. larvae | Rat-tailed maggot (Eristalis tenax) | Facultative myiasis | Onchocerca cervicalis | Cutaneous habronemiasis Facultative myiasis This is an example of facultative myiasis. Myiasis is the infestation of live animals with the larvae of dipteran flies. Facultative means that the larvae (maggots) are free-living but can invade damaged tissue (wounds, moist flesh). Obligatory myiasis occurs when the parasites require infection of the host to complete their life cycle. The main example of obligatory myiasis, screwworm (Cochliomyia hominivorax), has essentially been eliminated from North America, though occasional incursions occur through importation of animals. Screwworm infestation must be reported to state and federal authorities. Click here to see a screworm larvae and an adult screworm fly. Another differential in this cat is Cuterebra spp. These larvae are ingested or inhaled (or occasionally enter through wounds) and then migrate subcutaneously, causing swelling and sometimes pain. A breathing pore is often identified in the overlying skin. Click here to see a picture of a Cuterebra spp. larva. Culicoides spp. (biting midges or “no-see-ums”) irritate livestock and are associated with sweet itch (allergic dermatitis) in horses. Cutaneous habronemiasis in horses is caused by larvae of stomach worms and is characterized by granulomatous skin lesions with “sulfur crystals.” The rat- tailed maggot (Eristalis tenax) is associated with pseudomyiasis. These common larvae live in liquid feces and dairy farmers may mistakenly assume they came out of the cow (they didn't: Flies laid eggs in the manure). The maggots are the larvae of nonparasitic free-living flies. If you clean out barn gutters more frequently, you are less likely to see them. Onchocerca cervicalis is associated with dermatitis, fistulous withers, poll evil and uveitis in horses. Ref: Côté, Clinical Veterinary Advisor: Dogs and Cats, 4th ed., pp. 673-4. Image courtesy of Uwe Gille. 52. The protocol for cardiopulmonary resuscitation (CPR) is divided into 2 major components – Basic Life Support (BLS) that is initiated immediately, and Advanced Life Support (ALS) that follows. Which of the following correctly describes the major elements of Advanced Life Support? Needling of the GV acupuncture point to stimulate respiration | Administration of vasopressors and other medical therapy | Endotracheal intubation and ventilation with 100% oxygen | Cardiac compressions at 120 times/minute Administration of vasopressors and other medical therapy Advanced Life Support (ALS) involves the administration of medical therapies to patients in cardiac arrest. Epinephrine and/or vasopressin are given to produce peripheral vasoconstriction that will promote blood flow back to the heart, lungs, and brain. Reversal agents such as naloxone or atipamezole should be administered to those with opioids or alpha2 agonists on board. Medical issues such as hypoglycemia or hypokalemia are identified and treated. Cardiac compressions and ventilation that were started during the initial phase of CPCR, called basic life support, should be continued throughout the resuscitation attempt. Click here to download a free issue of the Journal of Vet Emergency and Critical Care with CPCR guidelines for animals. Refs: McCurnin’s Clin Textbk for Vet Techs, 8th ed. pp. 924-6. 53. Which one of the following diseases of cattle are transmitted by ticks? Image courtesy, Dr L Mahin. Coenurus cysts | Trypanosomiasis | Schistosomiasis | Polyneuritis bovi | Babesiosis Babesiosis Babesiosis is a red blood cell parasite that affects a wide range of domestic and wild animals. The main vectors of babesiosis are Rhipicephalus (Boophilus) species ticks. Babesiosis was eliminated from the U.S. in 1943 and is now a reportable disease. It took 40 years to eradicate B. bovis and B. bigemina by eliminating its tick vectors. Currently, the tick vectors persist only in a quarantine buffer zone between the U.S. and Mexico but reintroduction is a significant threat; the tick vectors have been detected periodically outside this buffer zone, and acaricide resistance is a growing issue for control. Click here for more information on the cattle fever tick and eradication program. Trypanosomiasis is spread by the tsetse fly. Refs: Smith, Large Animal Internal Medicine, 6th ed. p.1047. Image courtesy, Dr L Mahin. 54. Dr. Smith selects gabapentin as a medication for a 12-year-old boxer dog with osteoarthritis, who is in chronic pain. The dose is 10 mg/kg PO twice daily. The dog weighs 30 kg. Available gabapentin tablets are 100 mg/tablet. How many tablets are required to treat this dog for one week? 96 tablets | 14 tablets | 42 tablets | 21 tablets | 28 tablets 42 tablets 42 tablets of gabapentin are required to treat this dog for one week. Here's the calculation: 10 mg/kg x 30 kg = 300 mg/dose x 2 doses/day = 600 mg/day 600 mg/day x 7 days = 4200 mg/100 mg/tablet = 42 tablets 55. A three-year-old Quarter horse is presented that has collapsed in the last 30 minutes. The mare is recumbent and weak, with respiratory stridor, muscle fasciculations, sweating, prolapse of the third eyelid and her lips pulled back spasmodically, like a grin. The horse is hydrated and has had no recent history of colic. During examination, she improves markedly and within 60 minutes stands and appears almost normal again with no treatment. What blood chemistry abnormality during an episode might be expected for the condition at the top of the differential diagnosis list? Hypocalcemia | Elevated creatine kinase | Hyperkalemia | Na:K ratio less than 24:1 | High sodium Hyperkalemia Expect hyperkalemia during an episode of hyperkalemic periodic paralysis (HYPP). Remember: high potassium slow the heart. Usually (but not always) see INCREASED potassium greater than 5 mEq/L (normal 3-5) during an episode. Can see bradycardia, progressing to cardiac standstill and death. Rx involves 5% Na bicarbonate IV (helps get K+ back into cells by pulling H+ out of cells) and 23% Ca borogluconate IV to stabilize the heart. Dextrose IV may support movement of K+ into cells and diuresis (esp. if you don't have bicarbonate on the truck). Foals in laryngospasm with respiratory distress may need tracheotomy. Long-termTx involves acetazolamide. Onset of signs unpredictable, and many triggers: diets high in POTASSIUM (greater than 1.1%, e.g.: alfalfa hay, molasses, electrolyte supplements, kelp-based supplements). Fasting, anesthesia, heavy sedation, trailer rides, and stress can precipitate clinical signs. Refs: Smith, Van Metre, and Pusterla's Large Animal Internal Medicine, 6th ed., pp. 1427-30. 56. Classical swine fever (CSF) is also commonly known by what other name? Border disease | African swine fever | Erysipelas | Porcine dermatitis and nephropathy syndrome | Hog cholera Hog cholera Classical swine fever (CSF) is also known as hog cholera. CSF is a REPORTABLE disease, capable of causing devastating epidemics, particularly in countries like the U.S. which are free of the disease and do not vaccinate (so, entire population is at risk). CSF is characterized by fever, septicemia, and hemorrhagic lesions; DDX includes erysipelas, salmonella, PDNS, PMWS. Caused by a virus related to bovine viral diarrhea (BVD) in cows and border disease (in sheep, cows, pigs). African swine fever (ASF) is clinically INDISTINGUISHABLE from hog cholera/CSF but is caused by an unrelated DNA virus. Ref: Bassert, Beal and Samples, McCurnin's Clinical Textbook for Veterinary Technicians, 9th ed. p. 713. 57. How do you measure the quantitative immune response to a virus? Serum titer | Neutrophil count | Serum amyloid A | Lymphocyte count | Polymerase chain reaction Serum titer A serum titer (serology) measures the amount of antibody specific to a particular antigen (or virus) within the blood. It is the most specific way to measure an immune response to a particular virus. Polymerase chain reaction (PCR) looks for particular DNA; it can determine the presence of a virus but NOT the animal’s immune reaction to that virus. Changes in the lymphocyte count, neutrophil count, and serum amyloid A all can occur with viral infections but are very nonspecific and are not quantitative. Refs: Tizard I (2013). Veterinary Immunology, 9th ed. 58. A 15-year-old Arabian gelding is presented for recheck after one week of treatment for an intra- abdominal Corynebacterium pseudotuberculosis abscess. The horse has been on procaine penicillin (20,000 Units/kg IM BID), rifampin (5 mg/kg PO BID), and flunixin meglumine (1.1 mg/kg PO up to BID for fever). Clinically the horse has been improving with only two low-grade fevers in the last week and has already perked up a little. Today the physical exam parameters are within normal limits, though the horse remains a body condition score of 3+/9. Click on the labwork icon to review the hematology and blood chemistry results. Without performing further diagnostics, what would be the top differential for this horse’s anemia? Rifampin-associated hematuria | Immune-mediated hemolysis secondary to flunixin meglumine | Spread of the infection to the spleen | Anemia of chronic disease | Bone marrow suppression due to penicillin Anemia of chronic disease This history, clinical signs, and type and degree of anemia are most consistent with anemia of chronic disease. Anemia of chronic disease is the most common type of anemia identified in veterinary medicine. Chronic inflammation, infection, or neoplasia leads to leukocytic production of cytokines that causes decreased erythrocyte survival time, iron availability, and bone marrow response. This non-regenerative anemia can be mild to moderate. Usually resolves as underlying disease is treated. Flunixin meglumine has not been associated with immune-mediated hemolysis. Rifampin can turn urine red, but does not cause hematuria. Penicillin has not been associated with bone marrow suppression. Refs: Robinson and Sprayberry, Current Therapy in Eq Med, 6th ed. 59. A pair of heifers from a feedlot are presented. They are part of a group of 12-month-old heifers and steers that entered the feedlot three weeks ago. A steer died unexpectedly three days ago. Now, the two heifers are depressed and ataxic, with coughing and some dyspnea. One has opisthotonos and appears to be blind. The other is stiff and lame with a swollen hock. What disease is at the top of the differential diagnosis list? Infectious bovine rhinotracheitis | Thromboembolic meningoencephalitis | Hemagglutinating encephalomyelitis | Polioencephalomalacia | Listeriosis Thromboembolic meningoencephalitis History of recent feedlot entry, high fever, respiratory/CNS/Septic joint signs all point towards thromboembolic meningoencephalitis (TEME) caused by Histophilus somni (formerly Haemophilus somnus). Treatment is with antibiotics approved for this organism. Prognosis is good if treated very early, but not as favorable once musculoskeletal and/or neurologic signs appear. Prognosis is grave once animal is recumbent. Polioencephalomalacia (PEM) can present with identical neurologic signs so treatment with thiamine is also prudent. Do not expect obvious respiratory/fever signs with PEM. Ref: Smith, Van Metre, and Pusterla’s Large Animal Internal Medicine 6th ed. pp. 1069-70. 60. A three-day-old, 110 pound (50 kg), Holstein heifer calf with a history of watery diarrhea for 36 hours is presented in lateral recumbency. She quite thin and barely responds to examination. The mucous membranes are pale and dry, bluish in color, and capillary refill time is 4 seconds. Her eyes are sunken, and a skin tent remains in place. Which of the following is the best therapy for this calf? 5% dextrose and calcium | Intragastric fluids with hypertonic saline | Intravenous fluids and sodium bicarbonate | Subcutaneous lactated Ringer's Intravenous fluids and sodium bicarbonate Intravenous fluids with sodium bicarbonate must be given immediately for this calf to survive. She likely has a bacterial enteritis and may also be septic. Calf diarrhea is often a hypersecretory diarrhea that results in loss of huge volumes of fluid and bicarbonate. Dehydration and metabolic acidosis are life-threatening in severe cases. Isotonic bicarbonate at 100 ml/kg is recommended as the initial fluid volume, given over four to six hours. Dextrose is added as many calves are hypoglycemic. Once the calf is stronger and able to suckle, oral fluids alternated with milk are given in addition to maintenance fluids given intravenously. Refs: McCurnin’s Clin Textbk for Vet Techs, 8th ed. pp. 748-9, 892-3. 61. Which one of the following choices is the correct description of guttural pouch mycosis? Fungal infection of the guttural pouch | Air trapped in the guttural pouch | Viral infection of the guttural pouch | Bacterial infection of the guttural pouch Fungal infection of the guttural pouch Guttural pouch mycosis is a fungal infection of the guttural pouch, often caused by Aspergillus spp. Epistaxis is the most common clinical sign. Guttural pouch empyema is a bacterial infection of the guttural pouch. It is frequently associated with strangles caused by Streptococcus spp. Guttural pouch tympany is swelling of the pouch(es) caused by trapped air, usually found in young horses. Refs: Bassert and Thomas, McCurnin's Clinical Textbook for Veterinary Technicians, 8th ed. pp. 722-4. 62. Which one of the following conditions is defined as a disorder of the heart muscle itself? Subaortic stenosis | Sick sinus syndrome | Congestive heart failure | Cardiomyopathy Cardiomyopathy Cardiomyopathy is a disease of the heart muscle, it can be primary or secondary. Refs: Bassert and Thomas, McCurnin's Clinical Textbook for Veterinary Technicians, 8th ed. p. 688, Blackwell's 5 Minute Consult: Canine and Feline 4th ed. pp. 206-17, Saunders Comprehensive Veterinary Dictionary, Blood, Studdert and Gay, 4th ed. 63. A five-year-old Jack Russel terrier is coming in as an emergency with possible paraphimosis. What kind of problem is this? Eyeball popped out of socket | Mismating | Uterine proplase | Penis cannot retract into prepuce Penis cannot retract into prepuce When the penis cannot retract into the prepuce, the problem is called paraphimosis. A proptosis means "popping out or protruding." The most common emergency proptosis you will see in practice is a ocular proptosis (sometimes called an eye prolapse, too). Follow these links to see a proptosed eye in a dog and a severely proptosed eye in another dog. Ref: The Merck Veterinary Manual online edition. 64. A one-year-old Angus cow, shown in the image below, is presented with multifocal, circular, alopecic, and crusty lesions, mainly affecting the head and neck. Three other cows have also recently developed similar skin lesions. Dermatophytosis is the suspected problem. Which one of the following choices is the primary dermatophyte in cattle? Microsporum nanum | Trichophyton mentagrophytes | Trichophyton verrucosum | Microsporum gypseum | Microsporum canis Trichophyton verrucosum Trichophyton verrucosum is the primary dermatophyte in cattle. Microsporum canis is commonly found in dogs and cats, and Trichophyton mentagrophytes in rodents. Ref: Scott's Color Atlas of Farm Animal Dermatology, 1st ed. pp. 17-19. Image courtesy of Dr. Patrick Hensel. 65. A seven-year old Jersey cow who calved last night is down this morning. There have been no behavioral changes and she has been eating and drinking normally prior to this episode. She is weak with flaccid muscle tone. Udder examination shows normal milk; CMT is normal, cow has normal tail tone and there is no tenesmus. She vocalizes normally. No retained placenta, obvious infection or calf found on uterine palpation. The most likely presumptive diagnosis is? Septic mastitis | Septic metritis | Grass tetany | Rabies | Milk fever Milk fever The preferred response is MILK FEVER because of the recent history of calving in a dairy cow, with NO evidence of septic infection in uterus or udder. Although RABIES should always be considered in the DDx of any down cow, it's not common. Lack of behavioral changes, tenesmus and presence of normal tail tone and vocalization make rabies less likely as a primary diagnosis. Grass tetany is not likely because it presents with rigid paralysis. Ref: The Merck Veterinary Manual online edition. 66. A dead 18-month-old Jersey heifer had a 4-week history of poor appetite, unthriftiness, diarrhea and recently, submandibular edema. On necropsy small umbilicated nodules 1-2 mm in diameter are visible in the abomasum. Click here to see image Click here to see image What is the diagnosis? Anaplasmosis | Coccidiosis | Eosinophilic gastritis | Ostertagia infestation | Haemonchus infestation Ostertagia infestation An abomasum with nodules (i.e.: a cobblestone or "Moroccan leather" appearance) is pathognomonic for Ostertagia. Osteragia is one of the three stomach worms of cattle, along with Haemonchus (2.5 cm), and Trichostrongylus (5mm). Follow this link to see a Merck image of Ostertagia lesions in an abomasal lining. Worms remain dormant in the gastric glands, emerging in the spring. Coccidiosis is a hemorrhagic diarrhea of calves, usually younger than in this question, expect more tenesmus, frank blood. Anaplasmosis presents as an anemia, with fever and icterus. Ref: Smith, Van Metre, and Pusterla’s Large Animal Internal Medicine, 6th ed., pp. 1503-04. 67. A mature goat presents with the problem seen in this image and positional nystagmus. The goat is also febrile, lethargic, and circling to the left. Which one of the following choices is the best treatment for the presumptive diagnosis? Click here to see image Click here to see image Specific botulinum antitoxin, NSAIDs, and B-lactam antibiotics | Clarithromycin (PO, bid) and rifampin | Ponazuril (PO, 28 days), sulfadiazine with pyrimethamine | Thiamine (tid, IV 1st dose, then IM), dexamethasone and supportive care | High dose oxytetracycline or penicillin High dose oxytetracycline or penicillin Treat this likely case of listeriosis with HIGH DOSES of tetracyclines or penicillin. Isolate the goat and take it off silage, +/- IV fluids. Some of the signs are subtle in this image, but classically look for unilateral signs of brainstem and facial nerve dysfunction—swelling of cheek due to accumulation of food in the buccal space, drooling, ptosis (lowered eyelid), drooping ear, collapsed nostril, flaccid lip—ON AFFECTED SIDE. Can see deviated muzzle with lip pulled to OPPOSITE side of lesion. Follow this link to see a Merck image of facial nerve paralysis with obvious lip deviation AWAY from affected side in a horse. Typically listeriosis presents as encephalitis (depressed, fever) with unilateral limb signs (circling, leaning) and unilateral cranial nerve signs as discussed above. Can see sporadic ABORTIONS with listeriosis at any time in gestation, though more common in 3rd trimester. Listeria is ZOONOTIC. Highest risk is handling aborted fetus or performing necropsy of septicemic animal. With polioencephalomalacia, think opisthotonos, headpressing, blind, 'star-gazing' (dorso-medial strabismus via thiamine deficiency). Caprine arthritis encephalitis (CAE) presents as a polyarthritis in adults, progressive paresis in kids. Refs: Smith and Sherman, Goat Medicine 1st ed. pp. 141-43 and Smith, Van Metre, and Pusterla’s Large Animal Internal Medicine, 6th ed., pp. 969-70. 68. Salmonella typhimurium is associated with what condition in pigs? Swine dysentery | Rectal stricture | Secondary to intestinal spirochetosis | Intussusception | Rectal prolapse Rectal stricture Salmonella typhimurium is associated with rectal strictures in growing pigs. Caused by an ulcerative proctitis that damages rectal tissue. Can see large numbers of cases. Can see sporadic rectal strictures as a sequelae to rectal prolapse. Follow this link to see a Merck image of rectal prolapse in a sheep. Intestinal spirochetosis is a post-weaning diarrhea seen in the ABSENCE of Brachyspira (Serpulina) hyodysenteriae (swine dysentery), but similar in presentation to it. This syndrome is being recognized more frequently worldwide. Refs: Ogilve, LA Int Med, pp. 80-4. 69. An 11-month-old horse was recently euthanized after an acute onset of severe colic. On necropsy, it appears that the horse died from a strangulating volvulus of the small intestine. During completion of the necropsy, tiny, sand-like gritty particles can be felt in the wall of the colon as an incidental finding. What is the cause of these gritty particles? Peyer's patches | Sand in the intestinal crypts | Hypobiotic small strongyles | Encysted Oxyuris equi larvae | Villonodular lymphoid tissue Hypobiotic small strongyles Small strongyles (cyathostomes) develop in the wall and lumen of the intestine. Larvae in the wall of the intestine may undergo hypobiosis (arrested development) and can remain dormant for long periods. On necropsy, the hypobiotic larvae can sometimes be felt in the intestinal wall-they feel like tiny gritty nodules the size of sand grains (half a mm). Click here to see encysted hypobiotic cyathostome larvae in the colon of a horse. Cyathostomes in the lumen of the intestine are easily killed by anthelmintics (avermectins), but hypobiotic larvae in the intestinal wall are much more difficult to eliminate. A few small strongyles do not cause problems. But heavy infestations of hypobiotic larvae may cause problems when the encysted larvae leave the gut wall en-masse in late winter and spring, causing sudden weight loss, colic and diarrhea (esp. in young horses). Ref: Smith, Van Metre, and Pusterla's Large Animal Internal Medicine, 6th ed., p. 777-78, 1672. 70. Which one of the following phrases correctly describes bradykinin? A vital enzyme for protein digestion | An inflammatory mediator | A protein involved in hemoglobin production | It inhibits the clotting cascade | A substance that slows heart rate An inflammatory mediator Bradykinin is an important part of the inflammation response. Refs: The Merck Veterinary Manual online edition. Bassert, Beal and Samples, McCurnin's Clinical Textbook for Veterinary Technicians, 9thed, pp. 801 71. Which one of the following may be seen in a blood sample from an excited horse, and is associated with epinephrine release? Lymphocyte count is low | Decreased neutrophil count | Spurious anemia | Reactive thrombocytosis | Increased hematocrit Increased hematocrit Increased hematocrit due to splenic contraction. As part of epinephrine-mediated "flight or fright" response during excitement or exercise, the spleen contracts and releases RBCs into the circulation in response to increased levels of epinephrine. This increases the horse's ability to run faster or longer (to flee!). Excitement also leads to the following classic leukogram changes: neutrophilia, lymphocytosis. The spleen of a horse contains about one-third of the total volume of RBCs in circulation. Check out this good overview on erythrocytosis, courtesy of the Cornell eclinpath website. The classic cortisol-mediated "stress leukogram" includes: neutrophilia, lymphopenia, monocytosis, eosinopenia. Refs: Bassert, Beal and Samples, McCurnin's Clinical Textbook for Veterinary Technicians, 9thed. pp. 686. 72. A pig operation has an outbreak of sneezing, coughing and runny eyes in three to eight week old pigs. A few animals have bloody noses, and one has died. The following image is seen on necropsy. Which etiologies are thought to cause this disease? Click here to see image Click here to see image Bordetella bronchiseptica, Pasteurella multocida, ammonia | Actinobacillus pleuropneumoniae, Bordetella bronchiseptica, Aspergillus spp. | Dust, Bordetella bronchiseptica, Mycoplasma hyopneumoniae | Nitrogen dioxide, Pasteurella multocida, Mycoplasma hyopneumoniae | Actinobacillus pleuropneumoniae, Pasteurella multocida, nitrogen dioxide Bordetella bronchiseptica, Pasteurella multocida, ammonia This is atrophic rhinitis a disease with a complicated etiology - Bordetella bronchiseptica, Pasteurella multocida (types A, D), high ammonia, and dust are all thought to cause it. Disease divided into two forms: the NONprogressive form, caused by B. bronchoseptica is mild, transient, little effect on growth. The progressive form, caused by toxigenic P. multocida (types A, D) can cause severe, permanent damage and poor growth. Outbreaks usually follow either introduction of infected pigs or mixing pigs from different herds. Mycoplasma hyopneumoniae causes chronic, MILD respiratory disease. May become endemic, producing a persistent dry cough, retarded growth rate and occasional flare-ups of overt respiratory distress. Look for characteristic cranioventral lung consolidation. May predispose to a SECONDARY bacterial infection with Pasteurella. Actinobacillus pleuropneumoniae causes an acute, SEVERE pleuropneumonia. Look for suddenly dead pigs, fever as high as 107oF (41.5oC), anorexia, and reluctance to move. See respiratory distress (thumps), sometimes blood-stained frothy nasal, oral discharge. On necropsy see hemorrhagic, fibrino-necrotic pneumonia. Ref: The Merck Veterinary Manual online edition. 73. An 18-month-old, approximately 800-pound Angus heifer is presented for dystocia. Vaginal exam shows that the calf is alive in an anterior, dorsal presentation with the front legs presented and the head back. Following epidural anesthesia, several attempts are made to bring the head forward. In spite of using a lot of lubricant, there is not enough room in the pelvis and correction of the head position is unsuccessful. What is the best step to take next? C-section | Use a calf puller (winch) to gain leverage | Use more lube | Plank-in-the-flank | Fetotomy C-section A C-section should be performed to save this calf and its dam. A head back presentation is an indication of fetopelvic disproportion. Other clues include the heifer’s age (target age for first calf is 2 years) and size. Attempting to correct these is warranted, but after some failed attempts, continuing to try only tires the veterinarian and the animal. The “plank-on-the-flank” technique is used to correct uterine torsion. A calf puller/winch/block and tackle arrangement will give you more leverage on a calf, but if you cannot and should not try to use it on a calf with the head still back. A fetotomy is not appropriate for a live calf. Refs: Hopper, Bovine Reproduction, 1st ed., pp. 424-430 and Calving Management for Dairy Herds. 74. What neurologic signs might be seen with a T3-L3 Hansen's type I intervertebral disc disease? Cervical pain, extensor rigidity all four limbs, quadraparesis | Extensor rigidity in hindlimbs, flaccid paralysis in forelimbs | Forelimb and hindlimb flaccid paresis | Flaccid hindlimbs, loss of voluntary bladder control | Thoracolumbar pain, hindlimb ataxia Thoracolumbar pain, hindlimb ataxia Expect thoracolumbar pain, arched back, and hindlimb ataxia and weakness with Hansen's type I intervertebral disc disease (extrusion of nucleus pulposus through ruptured annulus fibrosis into spinal canal) in chondrodystrophic breeds. (i.e., Dachshund). Forelimbs are normal and a cutaneous trunci reflex may be absent from a cut off approximately two spinal cord segments caudal to the lesion. Reflexes in the hindlimbs are normal to increased due to decreased upper motor neuron (UMN) inhibition. Schiff-Sherrington phenomenon occurs with severe acute T3-L3 lesions and causes paradoxical UMN extensor rigidity in forelimbs and lower motor neuron (LMN) flaccid paralysis in hindlimbs. Ref: Cote, Clinical Veterinary Advisor, 4th ed., pp. 555-7, 1136-7 and De Lahunta, Veterinary Neuroanatomy and Clinical Neurology, 5th ed., pp. 271-4. 75. A rancher has some calves that have respiratory disease. He says he thinks they have "red nose". What disease is he referring to? Bovine respiratory syncytial virus | Infectious bovine rhinotracheitis | Parainfluenza-3 virus | Bovine viral diarrhea | Moraxella bovis Infectious bovine rhinotracheitis "Red nose" is the lay terminology for infectious bovine rhinotracheitis (IBR) caused by bovine herpesvirus-1 (BHV-1). BHV-1 is associated with a number of syndromes: rhinotracheitis, vulvovaginitis, conjunctivitis, abortion, enteritis, and encephalomyelitis. Prevention with immunization is usually effective. Moraxella bovis is a bacteria that causes infectious bovine keratoconjunctivitis, which can also occur in sheep and goats. Ref: Smith, Large Animal Internal Medicine, 6th ed. pp. 647-9. 76. Several deaths occur at a hog farm among rapidly-growing feeder pigs 4 to 6 months old. In the last 3 weeks, 2 pigs have died suddenly. The most recent one died last night, after a short bout of tarry black diarrhea. On necropsy, the small intestine is thin-walled and filled with clotted and unclotted blood. The large intestine has some tarry material in it. Which one of the following conditions is highest on your list of rule outs? Hemorrhagic bowel syndrome | Enterotoxigenic E. coli | Coccidiosis | Clostridium dificile | Clostridium perfringens Type C enteritis Hemorrhagic bowel syndrome Think of hemorrhagic bowel syndrome (HBS) in fast-growing feeder pigs with these signs. The cause of HBS is not clear, but intestinal volvulus is thought to be associated. Rule-outs include porcine proliferative enteritis and gastric ulcers. Can see sudden death without diarrhea. On necropsy, small intestine may be thin-walled, filled with clotted or unclotted blood. Large intestine may contain tarry material. Handling, fighting, piling, vigorous exercise, irregular feeding may predispose. PreventiveTx with tetracycline or bacitracin in feed. Here is a good summary article on HBS by Dr.Kent J Schwartz, Iowa State University. Don't confuse this condition in older pigs with Clostridium perfringens Type C enteritis which typically occurs in NEWBORN piglets 1 to 5 days old, and up to 3 weeks. Follow this link to see the hemorrhagic intestines of Clostridium perfringens. Refs: The Merck Veterinary Manual online edition and the Iowa Swine Disease Manual 4th edition (online), Neumann, Ramirez and Schwartz eds., Hemorrhagic Bowel Syndrome (HBS). 77. A guinea pig is presented with inappetence, oculonasal discharge, and dyspnea. The clients also have a pet rat, and a pair of rabbits, and these animals all have frequent contact with each other. Which of the following agents is the most likely cause? Bordetella bronchiseptica | Dromaius novaehollandiae | Hymenolepis diminuta | Clostridium difficile | Staphylococcus aureus Bordetella bronchiseptica Rabbits can be subclinical carriers of Bordetella bronchiseptica, which is highly pathogenic in guinea pigs. Because of this cross- species pathogenicity, rabbits and guinea pigs should not be kept together as pets. Hymenolepis diminuta is a dwarf tapeworm in gerbils. It is possibly zoonotic to humans. Ref: Quesenberry and Carpenter, Ferrets, Rabbits, and Rodents Clinical Medicine and Surgery 3rd ed. p. 206. 78. A five-year-old boxer is presented for routine vaccinations. The owner points out a small, smooth mass on the right ear. Fine needle aspiration cytology is shown below. Which one of the following choices is the most likely diagnosis? Histiocytoma | Bacterial infection | Degranulating mast cell tumor | Lymphoma | Aural hematoma Bacterial infection Bacterial infection. There are both intracellular and extracellular bacteria present in this cytology. The abscess should be drained and the animal placed on appropriate antibiotics. Mast cell tumors are round cell tumors with basophilic granules that are smaller than bacteria. Ref: Cote, Clinical Veterinary Advisor-Dogs and Cats, 3rd ed. pp. 5-6. Image courtesy, Nottingham Vet School. 79. Increased intraocular pressure is associated with which one of the following conditions? Cerebellar infarction | Glaucoma | Entropion | Corneal ulcer | Cherry eye Glaucoma Glaucoma is characterized by increased intraocular pressure. Refs: Bassert and Thomas, McCurnin's Clinical Textbook for Veterinary Technicians, 8th ed. pp. 702-3, Table 19-19. 80. Which kind of lesions are visible in the cut surface of the tongue of this cow with wooden tongue? Click here to see image Click here to see image Calcifications | Parasitic cysts | Viral vesicles | Tumors | Granulomas Granulomas These are the granulomas of wooden tongue, caused by Actinobacillus lignersi (remember LIGnersi-- LIGNIN is what makes WOOD). Seen mostly in cattle, but also in sheep, horses, pigs, and dogs. Look for hard swollen tongue, with pyogranulomatous lesions in SOFT tissues of head, neck, limbs. Follow this link to see a Merck image of A. lignersi granulomas in the cut surface of a tongue. Follow this link to see Merck image of A. lignersi swelling in SOFT TISSUE of a cow's neck. Actinomyces bovis is the cause of lumpy jaw in cattle. Remember " MY lumpY " for lumpy jaw (i.e: MYces, lumpY). Follow this link to see a Merck image of lumpy jaw. Ref: Smith, Van Metre, and Pusterla's Large Animal Internal Medicine, 6th ed., pp. 1248-49. 81. Gossypol toxicity in swine or young ruminants is due to use of cottonseed meal in feed, causing multisystemic effects (cardiac, hepatic, reproductive, renal). What supplement might a farmer using cottonseed meal add to the feed to help combat gossypol toxicity? Iron salts | Vitamin D | Potassium salts | Vitamin E/selenium | Psyllium and B vitamins (thiamine) Iron salts IRON helps to inactivate free gossypol pigment. High intake of protein, calcium hydroxide, or iron salts appears to be protective in animals fed cottonseed meal-containing feed. The best action to avoid gossypol toxicity is to remove cottonseed meal from the feed. By the time see signs and confirm Dx, prognosis is poor because exposure is usually long-term and multi-organ damage is already done. No effective treatment. Refs: Osweiler's NVMS Toxicology pp. 345-6. 82. A 3-year-old female spayed Labrador retriever is presented in southern Alabama for a suspected snakebite. The owner thinks the bite may have occurred during their morning walk an hour ago. The dog appears clinically normal. The owner killed the snake and brought it for identification: it is an eastern coral snake. Which of the following abnormalities is most likely if envenomation happened? Hypernatremia, hemorrhagic diarrhea | Coagulopathy, severe swelling around the bite | Hemolysis, paralysis | Muscle rigidity, hyperkalemia | Seizures, hypoglycemia Hemolysis, paralysis Expect hemolysis, lower motor neuron paralysis, and mental depression with coral snake (Micrurus spp.) envenomation. Clinical signs can progress extremely rapidly (within minutes) to the point of respiratory paralysis, necessitating mechanical ventilation. But onset of clinical signs can be delayed up to 36 hours! In contrast, pit viper envenomation is most commonly associated with coagulopathy and rapid, severe swelling around the bite. Coral snake bites are far less common than those of pit vipers (rattlesnakes, copperheads, cottonmouths) due to their limited geographic distribution. Click here to see an image of an eastern coral snake. Think “Red-on-yellow, kill a fellow. Red-on-black, friend of Jack” to remember which snakes are poisonous. Click here to see a comparison of the geographic distribution (scroll down) of U.S. coral snakes vs. cottonmouths and copperheads. Rattlesnakes can be found almost anywhere in the U.S. Refs: Coté, Clinical Veterinary Advisor–Dogs and Cats, 4th ed. pp. 921-5. 83. Which one of the following choices is the best diet for a cat with idiopathic hepatic lipidosis? High quality protein, high calorie | Balanced carbohydrate, with 15% soluble fiber | Low fat, with 10% insoluble fiber | Low protein, acidifying | Low carbohydrate, high fiber High quality protein, high calorie A high quality protein and high calorie diet is preferred for the management of hepatic lipidosis. The cornerstone of treatment for idiopathic hepatic lipidosis is nutritional support, initially via nasogastric tube, and then often by esophagostomy tube with a high calorie, high quality protein diet unless hepatic encephalopathy is present (uncommon). High-quality protein means it contains the right amino acids in the right proportions to fit the nutritional needs of the cat while causing less stress to the fragile liver than poorer quality proteins. High-quality protein meat or fish protein is recommended. Soybean protein is a "vegetable exception," which also has high-quality protein. A veterinary recovery diet usually meets these requirements. Vitamin K1 is recommended to treat/prevent the coagulopathy of liver disease. The owner needs to be educated regarding feeding tube care, and warned of the need for follow up blood chemistry analyses to assess recovery. Typically see recovery in three to six weeks. Refs: Cote, Clinical Veterinary Advisor: Dogs and Cats, 4th ed., pp. 444-6. 84. What is the most common viral agent of diarrhea in horses and what age group does it typically affect? Parvovirus, six to eight months | Equine viral diarrhea, one to three weeks | Equine distemper, two to four weeks | Rotavirus, two to three months | Panleukopenia, four to six months Rotavirus, two to three months Rotavirus is the most commonly identified cause of diarrhea in foals from two days to five months (most commonly about two to three months). It causes osmotic diarrhea by damaging the enterocytes on the tips of the small intestinal villi. Transmission is via the fecal-oral route, and diagnosis is typically made using a commercial immunoassay kit designed for detection of human rotavirus. The virus is extremely contagious, and, since it is non-enveloped, resistant to most disinfectants. Phenolic disinfectants are commonly used. Prevention involves use of an inactivated vaccine in pregnant mares at months eight, nine, and ten of gestation to maximize passage of protective antirotaviral antibodies in colostrum. Coronavirus is an emerging viral cause of diarrhea in horses of all ages. The other viruses listed as answer options for this question either do not exist or do not cause disease in horses. Ref: Sellon and Long, Equine Infectious Diseases, pp. 181-85. 85. Which one of the following lists consists of ectoparasites only? Ancylostoma caninium, Strongyloides westeri, Taenia krabbei | Dirofilaria immitis, Trichuris vulpis, Fasciola hepatica | Spirocera lupi, Physaloptera spp, Toxocara canis | Demodex canis, Cheyletiella spp, Argas persicus Demodex canis, Cheyletiella spp, Argas persicus Demodex canis, Cheyletiella spp, and Argas persicus are all external parasites or ectoparasites. Demodex canis is the agent of demodectic mange in dogs. Cheyletilla spp are skin mites found in dogs, cats, and rabbits. Argas persicus, the fowl tick, is a type of soft tick. Refs: Bassert and Thomas, McCurnin's Clinical Textbook for Veterinary Technicians, 9th ed., pp. 406-7. 86. What disease process is evident in this image? Ruptured chordae tendinae | Persistent right aortic arch | Mitral stenosis | Endocardiosis | Ventricular septal defect Endocardiosis This is endocardiosis (degenerative valve disease). Note the nodular thickening of mitral valve margins. Degenerative valve disease is the most common cardiac disease of dogs, accounting for about 75% of all canine cardiovascular disease. Roughly 60% of affected dogs have myxomatous degeneration of the mitral valve. Older, small-breed dogs have a higher incidence and Cavalier King Charles Spaniels are prone. Don't confuse endocardiosis, a degenerative disease with infective endocarditis which is due to bacterial infection of the cardiac valves and endocardial muscle. Refs: Cote, Clinical Veterinary Advisor-Dogs and Cats, 3rd ed. pp. 315-6, 668-70. Image courtesy of Dr. Terri DeFrancesco, Copyright 2022, All rights reserved. 87. Which one of the following choices is the correct concentration of isotonic saline? 0.9% | 0.45% | 23% | 3% | 7% 0.9% 0.9% saline is isotonic to plasma. Its osmolality is very similar to plasma and so will not change the tonicity of the intravascular space. Sodium and chloride are present at a slightly higher concentrations in isotonic saline than in plasma. It has an acidifying effect because of the chloride levels. Saline is used most often for specific patients rather than as a replacement or reususcitation fluid. It is used for patients with metabolic alkalosis or hypochloremia (vomiting) and in some hyponatremic patients. Ref: Bassert, Beal and Samples, McCurnin's Clinical Textbook for Veterinary Technicians, 9thed. pp. 821-2. 88. A dog is presented with numerous small pinpoint hemorrhages on the mucous membranes of the mouth. What are these hemorrhages called? Petechiae | Suppuration | Hematomas | Epiphora | Ecchymoses Petechiae Small pinpoint hemorrhages on the skin or mucus membranes are called petechiae. It is found in diseases that cause problems with coagulation. Larger hemorrhages (1-2 cm diameter) are called ecchymoses. Click here to see a dog with petechia. Click here to see a dog with ecchymosis on the gums. Click here to see a dog with ecchymosis on the abdomen. Refs: Bassert and Thomas, McCurnin's Clinical Textbook for Veterinary Technicians, 8th ed. pp. 224. 89. Necropsy of a dog with a history of kyphosis, spinal pain, collapse and pyuria shows the following lesion in a T3-L3 vertebral section. What disease is this? Click here to see image Click here to see image Canine degenerative myelopathy | Focal meningomyelitis | Ischemic myelopathy | Leukoencephalomyelopathy | Diskospondylitis Diskospondylitis This is diskospondylitis, a vertebral infection from hematogenous spread of bacteria (or fungi) from another site. Typically seen middle-aged big breeds, especially German Shepherds, Great Danes. Usually caused by hematogenous spread of STAPH AUREUS from primary infection elsewhere. See bacteremia, pyuria in 50%. Blood cultures positive in 75%. Rx is very long term ABX (Ave. treatment is 4 to 6 mos). Relapses are not uncommon. Px good for most with mild to moderate clinical signs. Can be caused by Brucella canis. Recurrence is common with B. canis. Ref: Blackwell's 5-Minute Vet Consult Canine Feline, 4th ed. pp. 392-93. 90. A three-year-old Irish setter is presented with a history of lameness and swelling in the distal forelimb. A radiograph is shown below. What is the presumptive diagnosis? Click here to see image Click here to see image Panosteititis | Hypertrophic osteodystrophy | Osteochondrosis | Osteomyelitis | Osteosarcoma Osteosarcoma This is a classic image of osteosarcoma, a COMMON, aggressive bone tumor typically found in the appendicular skeleton, especially distal radius. Here is a memory aid for common locations of osteosarecoma - "Away from the elbow, near the knee." 90% have microscopic metastases to lungs by time of Dx (but less than 10% will have visible thoracic metastasis at time of diagnosis). Usually, does not cross joint (unlike osteomyelitis, which usually DOES cross joint). Look for soft tissue swelling, periosteal proliferation, sunburst periosteal reaction (33%), possible pathologic fractures. Ref: Blackwell's 5-Minute Vet Consult Canine Feline, 4th ed. pp. 1006-07. 91. What is the most common cause of feline lower urinary tract disease? Electrolyte imbalances | Uroliths | Chronic renal disease | Idiopathic cystitis | Ur

Use Quizgecko on...
Browser
Browser